« first day (40 days earlier)      last day (534 days later) » 
00:00 - 19:0019:00 - 00:00

12:02 AM
0
Q: Show that s=supA

SelfStudyI am wondering if my solution is correct Question: Let $A ⊆ \mathbb{R}$ be nonempty and bounded above, let $s\in \mathbb{R}$ have the property that for all $n \in \mathbb{N}, s+1/n$ is an upper bound for A. Show that s=sup A. My solution: We observe first that, for each $x \in A$ we have $x \...

Short title. Show that s=supA
0
Q: How can mathematics help to save the planet earth?

KhosrotashWhen I was teaching plastic decay formula in a class ,a deep question asked to me .someone asked "How can mathematics help to ,save the planet earth?" I made almost the same example $$m\overset{T}{\rightarrow} \frac{m}{2} \overset{T}{\rightarrow}\frac{m}{4} \overset{T}{\rightarrow}\frac{m}{8}......

Words such as help do not add information to titles. Please edit the title so that it better describes the specifics of your question. Do not hesitate to make it longer or include a formula if needed. More tips here. (autocomment)Normal Human 21 secs ago
 
0
Q: Badge proposal: Quality Poster

Caleb KleveterI have noticed that a lot of new SO users don't post vary quality posts. I was thinking that maybe if we had a badge to encourage good posting things might improve. Sure, we have the Curious/Inquisitive/Socratic, but I was think something a little harder that you have to get right from the get-go...

 
0
Q: Solve a problem using Markov chains

USS LimitzWe have the following problem: At the beginning of every year, a gardener classifies his soil based on its quality: it's either good, mediocre or bad. Assume that the classification of the soil has a stochastic nature which only depends on last year's classification and never improves. We hav...

Title contains problem. Solve a problem using Markov chains
0
Q: Annuity and Loan Repayment Question. Show the amount of Loan.

HeavyWeightA loan was taken out on 1 September 1998 and was repayable by the following scheme: The first repayment was made on 1 July 1999 and was £1000. Thereafter, repayments were made on 1 November 1999, 1 March 2000, 1 July 2000, 1 November 2000, etc until 1 March 2004, inclusive (note that the loan was...

 
12:30 AM
0
Q: creating a maclaurin series givin facts

Noah Dukeharti am working on a practice exam for my calculus 2 course and am working through the practice exam and i have come to a question i have no idea of how to start or where to go with it. here is the question: Knowing that (d^k/dx^k)(1-x)^-1=k!(1-x)^(-k-1), we can say the the maclaurin series of f(x)...

0
Q: Superposition principle

Mary StarWhen we have a differential equation and we apply the superposition principle, does it stand that the initial equation has a solution if and only if the subproblems have solutions? Or is it just if?

Welcome to Math.SE, Noah Dukehart. This site uses MathJax formatting of formulas. More tips here. (from a bot)Normal Human 20 secs ago
0
Q: Centres of Curvature

NotNotLogicalSuppose we have a space curve $r(t)=\left(x(t), y(t), z(t)\right)$ and we consider the curve of all centres of curvature of $r$. First, would this be the correct formula? $$r(t) + {1 \over \kappa(t)} N(t)$$ (where $N$ is the normal vector in the TNB-frame and $\kappa$ is curvature). And is there ...

Short title. Centres of Curvature
 
12:51 AM
0
Q: what does the volume of solid of revolution represent?

kadiI know that the area under a curve represents the displacement of an object, but what does the area enclosed by two curves and the volume of the solid formed by a revolution represent?

0
Q: Permutation and Combinatorics Problem

sarvagyaA function $G$ is defined on a set $S$ with size $k$ : $G(a_1,a_2,a_3,.....,a_k)$. $G(a_1,a_2,a_3,.....,a_k) = 1$ if and only if a convex polygon can be created by taking these $k$ elements as the side lengths. Otherwise $G(a_1,a_2,a_3,.....,a_k) = 0$. You are given the identity permutation over ...

 
1:11 AM
0
Q: Prove that the sum of two numbers is irrational

MatthewGoal: Prove that $5^{1/3}$+$7^{1/2}$ is irrational. Idea: We can prove this is irrational by supposing it is rational and finding a contradiction. So, $5^{1/3}$+$7^{1/2}$ = $p/q$ where p and q are integers that have no factors in common other than 1. The issue here is that I can not seem to find...

0
Q: Integral over a line in the complex plane (not a contour)

T. WelshForgive me if I'm suffering from a fundamental misunderstanding. I have a function where a is a real positive constant: $$ \frac{(-1 + \pi \sqrt s \cot( \pi \sqrt s ))*(-1 + \pi \sqrt{-a +s} \cot(\pi \sqrt{-a+s})}{4t(-a+s)}$$ This has an infinite number of discontinuities along the positive rea...

Consider adding a tag for a broader subject area to which the question belongs. Some of these tags might fit. (from a bot)Normal Human 21 secs ago
 
1:34 AM
0
Q: Fixed Point problem of walter Rudin

user120386Suppose $f$ is differentiable on $(-\infty , \infty)$. We say that $x$ is a fixed point if $f(x) = x$. If there is a constant $A$ such that $0 <A <1$ and $|f'(x)| \leq A$ for all real $x$. Then there is a fixed point $x$ such that $x = \lim_{n\to \infty} x_n$ , where $x_1$ is any arbitrary real ...

Title contains problem. Fixed Point problem of walter Rudin
0
Q: Frobenius method confusion?

Joey ZhangThe question is: We are supposed to find the first 4 terms in the power series. I was able to work through the question but the my answer is wrong, the correct answer is supposed to be the strange thing is that the answer I got was basically this but the coefficients are square roots of wha...

Short title. Title contains confus. Frobenius method confusion?
 
1:46 AM
0
Q: compute $E[X_1 |X_2]$; $E[S_n|X_1]$; $E[S_n |S_{n-1}]$

Alexlet $(X_n)$ i.i.d integrable random variables and $S_n=\sum_{i=1}^{n}X_i$ How can i compute $E[X_1 |X_2]$; $E[S_n|X_1]$; $E[S_n |S_{n-1}]$?

Short question. [compute $E[X_1 |X_2]$; $E[S_n|X_1]$; $E[S_n |S_{n-1}]$](math.stackexchange.com/q/1571619)
0
Q: Automorphism group of $G\times G$ has an element of order 2

sequence Given that $G$ is a non-trivial group, prove that Aut$(G\times G)$ has an element of order 2. At this time I lack good intuition for automorphism groups, so I would appreciate some hints about how the statement above be proved.

0
Q: What is math analysis?

HwishzohgOk so I know that math is math. But if you try to figure out analysis. Anal-lysis. Lysis means break down. So... Does math analysis... Break down Uranus? Why doesn't it break down Neptune too?

0
Q: Filled Julia set

user121955How would I find the filled Julia set for $f(z)=z^3$? I know it should be the filled unit circle, but I don't quite understand the math. This is what I have so far: Fixed points $z^3=z$ so $z=1,-1,0,\infty$ $f'(1)=3>1$--repelling $f'(-1)=3>1$--repelling $f'(0)=0<1$--super attracting $f'(\infty)=0<

Short title. Filled Julia set
0
Q: Box topology and product topology

IllustionistWhy is it the case that the box topology and product topology agree if we consider finite products?

0
Q: How to solve a problem with a variable in both the base and exponent on opposite sides of an equation

Anson SavageI am working on systems of equations in Pre-Calculus, and I presented the teacher a question that I had been wondering for a while. $x^2 = 2^x$ The teacher couldn't figure it out after playing with it for quite a while. What are some ways it can be solved algebraically? Of course it can be so...

0
Q: what is the least number n for O((logx)^4) = O(x^n)

Argonne I am not sure how to compare (logx)^4 with x^3 or x^4 to get the answer (either 3 or 4).

0
Q: How to calculate the integral \int_\sqrt[]{y/3}^1 exp(x^3) dx

ZiloricHow to calculate the integral $$ \int_\sqrt[]{y/3}^1 exp(x^3) dx $$

Short question. [How to calculate the integral \int_\sqrt[]{y/3}^1 exp(x^3) dx](math.stackexchange.com/q/1571637)
0
Q: Particular solution: (3+x) e^(-2x)?

33tedy''+4y'+4y = (3+x) e^(-2x) So I'm working with undetermined coefficient and figured out solution for the left side. But what is the particular solution for the right side? I tried these but they don't work as all terms cancel to zero: y = (Ax+B)xe^(-2x) y = (Ax+B)e^(-2x)

Welcome to Math.SE, 33ted. This site uses MathJax formatting of formulas. More tips here. (autocomment)Normal Human 21 secs ago
 
2:30 AM
0
Q: Help solving $U_{n+1}=(U_{n})^{2} (n+2)$

J.GudalI need help solving the recurrence relation: $U_{n+1}=(U_{n})^{2} (n+2)$, with $U(1)=2$. I've tried wolfram alpha, but something really horrible came up. The methods I've tried have just failed so I need some ideas please.

Words such as help do not add information to titles. Please edit the title so that it better describes the specifics of your question. Do not hesitate to make it longer or include a formula if needed. More tips here. (autocomment)Normal Human 21 secs ago
0
Q: Integral of (logx)^2e^x

Mayank ChaturvediHow to calculate integration 0 to infinite e^x*(log x)^2.I have tried putting x=log t but can't reach the solution.

This site uses MathJax formatting of formulas. More tips here. (from a bot)Normal Human 21 secs ago
0
Q: Probability question : Is this true?

Electro82 How to explain this: P(B|A)+P(B|(complement A) ) = 1 Can anyone help me? I can't see how to explain this. Thanks!

Words such as question are uninformative in titles. Please edit the title so that it better describes the specifics of your question. Do not hesitate to make it longer or include a formula if needed. More tips here. (autocomment)Normal Human 21 secs ago
0
Q: Are there any theorem about the linearization of PDE?

lanse7ptyI am a beginner of PDE, and surprise that some nonlinear equation will become a linear equation after variable substitution,for example. So, I am curious that are there general theory making equation become linear. If not, why we don't do so ? I want to know the difficult of this way.

Tagged pde, differential-equations. Tagged differential-equations but mentions "partial". Are there any theorem about the linearization of PDE?
0
Q: Understanding backpropagation

aceminerI was studying some neural networks back propagation from http://jeremykun.com/2012/12/09/neural-networks-and-backpropagation/ I did not catch how did he achieve this part. Could someone explain to me? Thanks!

Short title. Short question. Understanding backpropagation
0
Q: Choosing the best coordinate system for triple integrals?

Ibrahim AhmedI am working with double and triple integral in multi-variable calculus and have found that it is extremely useful to convert between different coordinate systems including: Spherical: Cylindrical: Polar: and Cartesian Well, I know the conversion values and how to transition variables over. M...

Welcome to Math.SE, Ibrahim Ahmed. This site uses MathJax formatting of formulas. More tips here. (autocomment)Normal Human 21 secs ago
1
Q: Actuarial Problem. (Policyholder).What is the probability that a new policyholder will have an accident within a year of purchasinag a policy?

DarioC Problem said: Suppose people can be divided into two classes: those who are accident-prone and those who are not. The statistics show that an accident-prone person will have an accident at some time within a fide 1-year period with probability 0.35, whereas this probability for a non-a...

0
Q: Existence of Galois extension

user117375Let $G$ be a finite group. Then there exist fields $L$ and $K$ such that $L$ is an extension of $K$ with Galois group $G$. I think that since $G$ is finite, then $G$ must isomorphic to a subgroup of permutation group $S_{|G|}$. I think that Galois extension has connection with symmetric group $S...

 
3:24 AM
0
Q: Finding Expectation and Variance of X1 and X2

MooseLet X1 and X2 be random variables such that E(Xi)=μi and Var(Xi)=αi^2. A. Find E(X1+X2) and E(X1-X2) in terms of the μ's and α's. B. Suppose that E(X1X2)=α. Find Var(X1+X2). When does it equal Var(X1)+Var(X2)? C. Assume that X1 and X2 are independent. Find Var(3*X1-2*X2). Please explain how t...

Welcome to Math.SE, Moose. This site uses MathJax formatting of formulas. More tips here. (from a bot)Normal Human 21 secs ago
 
-1
Q: Automatic dup closure of questions with nullpointerexception

MrsEdThere are nullpointerexception questions being asked under the java and android tags daily, they regularly get closed as dupes. http://stackoverflow.com/questions/tagged/nullpointerexception Can we have a bot that automatically closes questions with nullpointerexception or java.lang.NullPointer...

 
0
Q: What are the unknown angles and side lengths?

Atoosa I'm really confused by this and I'm not sure how to answer it so a detailed explanation on how to would be great, thank you!

0
Q: Having an issue with integral substitution

daniel furhangI'm currently reviewing for finals and I tried to solve this question, but the program I'm using tells me I got it wrong. Can someone tell me where I screwed up? this is the question- evaluate the indefinite integral of int/(sin2x)(cos2x+1)^(1/2)dx I used substitution to replace (cos2x+1) with '...

Welcome to Math.SE, daniel furhang. This site uses MathJax formatting of formulas. More tips here. (from a bot)Normal Human 21 secs ago
 
3:44 AM
0
Q: Find all positive integers(a,b,c,n) such that 2^n=a!+b!+c!

RIO DuttaI have found the solutions by a little calculation (2,3,5,7) and (2,3,4,5). But I don't know if there's any other solutions or not?

0
Q: Fourier transform of phase shifted cosine function

hariFourier transform of function $\cos(w_m-w_0)t$ is $\pi\left [ \delta (w-(w_m-w_0)) \right + \delta (w+(w_m-w_0)) ]$. What would be FT of $\cos((w_m-w_0)t+\phi)$?

0
Q: $\sum a_n $ absolutely convergent and $\sum b_n $ convergent $\implies \sum a_nb_n$ absolutely convergent.

AmartyaIs this true? $\sum a_n $ absolutely convergent and $\sum b_n $ convergent $\implies \sum a_nb_n$ absolutely convergent. I don't know how to proceed .Please help.

0
Q: Surface area of the part of the surface

MathematicsFind the surface area of the part of the surface given by $x=3y+z^2$ that lies between planes $y=0$, $y=z$, $z=0$ and $z=3$ Is this integral I have set-up correct?

0
Q: How do I find an $A_0$ and $A_n$ which meet the initial conditions of this heat equation?

JimmLet's say I have the heat equation $\frac {\partial u}{\partial t} = k\frac {\partial^2 u}{\partial x^2}$, $0 \lt x \lt L$, $t \gt 0$, subject to the boundary conditions $\frac {\partial u}{\partial x}(0, t) = 0$, $t \gt 0$ $\frac {\partial u}{\partial x}(L, t) = 0$, $t \gt 0$, $L$ of course b...

Tagged pde, differential-equations. Tagged differential-equations but mentions "partial". How do I find an $A_0$ and $A_n$ which meet the initial conditions of this heat equation?
 
4:02 AM
0
Q: Ethics - Redirecting homework solution seeking questions to a website built for that purpose

Vlad KMy question is: Is it acceptable to notify users who ask "solve this for me" questions of a site built specifically for that purpose? What I would like to do is post a comment on "solve this problem" questions referencing how to get help with homework on PSE the right way, and referencing the si...

 
4:12 AM
0
Q: surface area of the part of the given surface

MathematicsFind the surface area of the part of the surface given by $x=3y+z^2$ that lies between planes $y=0$, $y=z$, $z=0$ and $z=3$ Is this integral I have set-up correct? $\displaystyle \int_{0}^{3} \int_{0}^{z} \sqrt{10+4z^2} dy dz$ Please look into it

0
Q: Is this a sufficient proof of a math contest problem?

Kenny DuranProblem: If a,b,c,d are real, prove that $$a^2+b^2=2$$ $$c^2+d^2=2$$ $$ac=bd$$ Is true if and only if $$a^2+c^2=2$$ $$b^2+d^2=2$$ $$ab=cd$$ My proof is as follows: Note that each of the second set of equations nearly corresponds to the lengths of the sides of a right triangle. The eq...

Title contains problem. Tagged proof-verification. Is this a sufficient proof of a math contest problem?
 
4:35 AM
0
Q: The lengths of the sides of a triangle form an arithmetic progression.

Angelo MarkThe lengths of the sides of a triangle form an arithmetic progression. Prove that the radius of the inscribed circle is one third of one of the heights of the triangle.

0
Q: Calculating integral - I have to calculate the following integral

zerosI need some help to solve this integral $$\int \frac{dx}{ \sqrt[]{-x^{2}-3x -3} } $$

 
4:57 AM
0
Q: Aut($\mathbb{Z}_2\times \mathbb{Z}_2)\cong \mathbb{Z}_6$

sequenceWould it be correct to prove this statement as follows? Since a generator must go to a generator under an automorphism, $(0,0)$ must always be sent to $(0,0)$ under any $\phi\in$ Aut($\mathbb{Z}_2\times \mathbb{Z}_2)$. Now, for Out($\mathbb{Z}_2\times \mathbb{Z}_2)$, the only possible permuta...

0
Q: Describe union of set

RatonLet S describe the set $\bigcup_{n=1}^\infty A_n$ where $A_n = \{(x, y) \in \mathbb{R}^2 | y-x^{2n} \geq 0 \}$ I understand that S describes the set of points "inside of the parabola y = $x^2$, but am not sure how to prove this. Any ideas?

0
Q: A truly beautiful integral, but no solution yet...

0.5772156649...$$I=\int_0^1 \frac{\sin(\ln^4 (1-x))}{x}$$ What is the closed-form evaluation of this integral? I honestly do not have a single clue how to move on from here. (There is no application to the integral, but it is out of curiosity.)

Words such as beautiful are uninformative in titles. Please edit the title so that it better describes the specifics of your question. Do not hesitate to make it longer or include a formula if needed. More tips here. (from a bot)Normal Human 21 secs ago
0
Q: Help you acquire healthy brain functions

ryaneatonThese are proven elements which can enhance cognitive abilities within short period of time. You might have to understand at this point. Here comes the role of the cognitive enhancing properties. I guess it's cool that there are so many hordes out there with . Life seems to be incomplete without ...

Welcome to Math.SE, ryaneaton. Words such as help do not add information to titles. Please edit the title so that it better describes the specifics of your question. Do not hesitate to make it longer or include a formula if needed. More tips here. (from a bot)Normal Human 21 secs ago
 
5:24 AM
0
Q: Prove $v$ is harmonic and $\lim_{r \uparrow 1} v(re^{i\theta}) = 0$

Due_DateProve that if $v(z) = \mathrm{Im}[(\frac{1+z}{1-z})^2]$, then $v$ is harmonic and $\lim_{r \uparrow 1} v(re^{i\theta}) = 0$. Explain why this does not contradict the maximum principle.

0
Q: If $y=\sum_i\frac{x_i}{\sigma_i^2}$ then is $\frac{\partial y}{\partial x_i}=\sum_i\frac{1}{\sigma_i^2}$ or is it equal to $\frac{1}{\sigma_i^2}$?

BLAZEI thought that the former I correct, but then I read this as part of certain proof: Should the sigma be omitted on taking the partial derivative?

 
5:51 AM
0
Q: Does $\sum f_n$ converge uniformly on an interval $[a, b]$

GregSuppose $f_n$ are continuous real valued functions on $[a, b]$ such that $\sum_{n=1}^\infty f_n$ converges, does it follow that the sequence $a_N =\sum_{n=1}^N f_n$converges uniformly?

Short question. [Does $\sum f_n$ converge uniformly on an interval $[a, b]$](math.stackexchange.com/q/1571790)
0
Q: What's the pointwise convergence of $\{nx^n(1-x)\}$?

Kenneth.KIs $\{nx^n(1-x)\}_{n=1}^{\infty}$ pointwise convergent on $\mathbb{R}$? If it is, what's the pointwise limit?

0
Q: Substitution principle example?

user139985 I have to know the substitution principle for an upcoming exam. This is the definition given in my book (sorry it's split up all weird). I understand the notation I believe. However, I am having a very difficult time understanding what this might look like. Can anyone give me a simple example t...

0
Q: Find the domain of $\sec^{-1}(\frac{2+x}{1-x})^{\frac{1}{x}}$

Vinod Kumar PuniaFind the domain of $\sec^{-1}(\frac{2+x}{1-x})^{\frac{1}{x}}$ Since the domain of definition of $\sec^{-1} x$ is $x\leq-1$ or $x\geq 1$. So $(\frac{2+x}{1-x})^{\frac{1}{x}}\leq -1$ or $(\frac{2+x}{1-x})^{\frac{1}{x}}\geq 1$ I am stuck here.Please help.

 
6:13 AM
0
Q: Show that if $E$ is a finite dimensional subspace of $l_p$, there exists an integer $m$ such that $\| P_m(x) \| \geq (1 - \dfrac{1}{n}) \| x \|$

IdonknowShow that if $E$ is a finite dimensional subspace of $l_p$, there exists an integer $m$ such that $$\| P_m(x) \| \geq (1 - \dfrac{1}{n}) \| x \|$$ where $x \in E$ and $P_m$ is a projection map from $l_p$ onto a subspace generated by its first $m$ basis vectors. The question comes from the proo...

Tall formulas in titles break the layout of question lists. Please replace \dfrac with \frac in the title. (from a bot)Normal Human 21 secs ago
 
6:44 AM
0
Q: Express the real root of $x^3-3x+7$ using radicals

Makoto K.I want to express the real root of $x^3-3x+7$ using radicals. My attempt is contained in the answer below.

0
Q: Need help proving any subgroup and quotient of a nilpotent group is nilpotent?

user264885$G$ is the direct product of its Sylow subgroups $P_i$. Then if $H \le G$, $P_i \cap H \le H$. I'm stuck now on how to proceede to find a terminating central series for $H$. Also, I know $P_i H / H \le G/H$ so I'm assuming proving the above will prove that the quotient is nilpotent too?

 
7:01 AM
0
Q: Decimal part of non-integer

Rayan AhmedA reciprocal of p's non-integer decimal part is equal to $p+1$ where $p >0 $. Can you please explain this to me.

Short title. Short question. Question contains please. Decimal part of non-integer
0
Q: Conditional Probability calcualtion

clarksonIn the following BBN network, 1)what is meant by P(Martin Late|train strike,Norman Late)? Does this mean probability of martin Late given that Train Strike And Norman is Late? 2) what is meant by P(Martin Late, Norman Late|Train Strike)? Is this the probability that Martin and Norman are l...

 
7:37 AM
0
Q: An inclusion exclusion type problem

Aloysius GodinhoIn a group of 265 people, 200 like singing, 110 like dancing and 55 like painting. 60 like both singing and dancing, 30 like both singing and painting and 10 like all three activities. How many persons like only dancing and painting. So let $S$ be the set of singers, $D$ the set of dancers and ...

Title contains problem. An inclusion exclusion type problem
 
8:01 AM
0
Q: How many hours I spend on SO

VR46Is there a way to find how many hours I spent on SO. Also who has spent most number hours on SO(probably John Skeet ;)). Is there such option available. If No then adding will be good. What you guys say?

 
0
Q: Module of vector bundles

6666If E→M and F→M are two vector bundles over manifold M, then $C^{\infty}$(E,F) is a $C^{\infty}$(M) module. I am confused about this claim for the operation of module. To f$\in$$C^{\infty}$(E,F) and g$\in$$C^{\infty}$(M), how could g operate on f? f is from E to F while g is from M to R.

 
0
Q: Keyboard problem when editing

RomaszI'm facing strange problem while editing questions/answers. My Up key works only up to the beginning of the paragraph where cursor is present (in other words it works up to first linebreak above the cursor). No matter if it's code or normal text. To go to upper paragraph, I've to use mouse, and ...

 
8:35 AM
0
Q: (1234)%6 = ((123%6)*10 + 4)%6 why this relation works?

ZANI know these relations, (a * b * c * d...)%m = ((a%m) * (b%m) * (c%m) * (d%m)......) %m and (a+b+c+d+........)%m = (a%m + b%m + c%m + d%m)%m But , how does this relation work? 1297 % 6 = ((129%6)*10 + 7)%6 129 % 6 = ((12%6)*10 + 9 ) %6 and so on ..... If I expand 129 % 6 ...

This site uses MathJax formatting of formulas. More tips here. (from a bot)Normal Human 29 secs ago
0
Q: $RSA$ cryptosystem with $e=2$

mattThere exists a $RSA$ cryptosystem with $e=2$ , where $e$ is the encryption exponent ?(In general $e>2$)

Short title. Short question. $RSA$ cryptosystem with $e=2$
0
Q: betweeen multiple logistic regression model and fitted response logistic function of full model.

Ron AldI have been trying to understand the betweeen multiple logistic regression model and fitted response logistic function of full model. I think log(pi/(1-pi)) is multiple logistic regression model. And estimated pi = exp(Xb)/(1+exp(Xb)) is fitted response logistic function. I read the book and I ...

This site uses MathJax formatting of formulas. More tips here. (from a bot)Normal Human 29 secs ago
 
8:58 AM
0
Q: Does $\sum_{i\neq j}$=$\sum_{i=1}^{n}\sum_{j=1}^{n}$?

Spencer IrelandI want to know if $\sum_{i\neq j}$=$\sum_{i=1}^{n}\sum_{j=1}^{n}$?

0
Q: find the distribution of method of moment estimate and the expectation of method of moment estimate

Sung Pang$X_{1}, ... , X_{n}$ $i.i.d$ with density function $$ f(x|Θ) = exp^{-(x-Θ)}, x >= Θ$$ I could find method of moment estimate of Θ. method of moment estimate of Θ : $$\hat Θ = \bar{X}-1$$ How can I find the distribution of MME ? I found MLE, distribution of MLE, MME but I can't find distribut...

Short question. Question contains please. A light bulb is connected as in the circuit shown below.
0
Q: "Invalid sequence" in Maple

LJRI tried to define a procedure in Maple as follows. removeAnElementInList:=proc(i, l) local r, j; r:=[]; for j from 1 to lengthOfList(l) do ( if(j<>i) then r:=append(r, l[j]) end if ) end do; return(r); end proc; But Maple returns an error: invalid sequence (at "then"). But I d...

0
Q: Complex analysis-Mean theorem

math.lvis the Mean theorem valid for complex analysis i mean if f is analytic in a convex region, can we alawys find a point c on the line segement between two points of this region(a,b) so that: f(a)-f(b)=f'(c)*(a-b) ??

0
Q: Prove this inequality for sufficiently large n

some oneProve that the above inequality holds for sufficiently large n. pi(2n) - 1.5 pi(n) >= O ( log n / (log log n )^2 ) Log n denotes to natural logarithm and Pi(n) is prime counting function.

This site uses MathJax formatting of formulas. More tips here. (autocomment)Normal Human 24 secs ago
0
Q: A question about the matching hats problem in probability

probabilitydoesntsuckThis is from my practice final exam. A variation with '8' instead of '7' can also be seen in an exam paper from MIT ocw probability course here: http://ocw.mit.edu/courses/mathematics/18-440-probability-and-random-variables-spring-2014 7 people throw their hats into a box and then randomly redi...

Words such as question do not add information to titles. Please edit the title so that it better describes the specifics of your question. Do not hesitate to make it longer or include a formula if needed. More tips here. (autocomment)Normal Human 25 secs ago
0
Q: Diophantine problem related to pythagorean triples: prove 2 expressions cannot both be perfect squares

Ameet SharmaGiven 2 primitive pythagorean triples with parameters as per: https://en.wikipedia.org/wiki/Pythagorean_triple#Generating_a_triple First triple has parameters $(m,n)$. Second has parameters $(r,s)$. Note that $m>n,r>s$ We define $g$ and $h$ according to these equations: $$g^2 = (mn(r^2-s^2))^2 ...

 
9:44 AM
0
Q: Having trouble computing this determinant, using multi-linearity,

User001The problem statement is: Show that there exists numbers $a$ and $b$ such that $$det (A + sxy^*)= a+bs$$ here $A$ is an $nxn$ matrix with real entries, and $x,y\in R^n$. I've been kind using brute force and using multi-linearity of the determinant several times, and the computations are gett...

0
Q: Show that $int (bd (A)) = \nothing $

mahdi moosazadehIf (S,T) be a topology space and $A$ be a subset of $S$ and $bd(A) = \overline{A} ∩ \overline{A^c} $ Show that $int (bd (A)) = ∅$ if and only if $bd (bd (A)) = bd (A)$

0
Q: Hausdorff dimension of a countable set

g.pomegranateI don't understand why the Hausdorff dimension of a countable set in $\mathbb{R}^n$ is $0$. Can someone please give me a hint? Thank you!

Questions tend to get more attention when they have a tag for a broad area of mathematics relevant to the question. Some of these tags might fit. (autocomment)Normal Human 22 secs ago
 
10:10 AM
0
Q: Fourier transform , of a function computation

MarussTI need help with this Fourier transform computation. F(w)=\int_{\infty}^{\infty} e^{-|x|+ix}e^{-iwx}$

0
Q: A doubt in Eberhard Zeidler's 1st volume on Quantum Field Theory

Rohan ShettyCan someone explain the proof of the result 7.131 on page 450 of Eberhard Zeidler's 1st volume on Quantum Field Theory?

Title contains doubt, hard. Short question. A doubt in Eberhard Zeidler's 1st volume on Quantum Field Theory
0
Q: Cardinality of the smallest subgroup containing two distinct subgroups of order 2

user118494$G$ is a finite group and $H_1$,$H_2$ are two disjoint subgroups of order $2$. $H$ is the subgroup of smallest order that contains both $H_1$ and $H_2.$ What is the cardinality of $H$ $?$ $A.$ always $2$. $B.$ always $4.$ $C.$ always $8.$ $D.$ none of the above. No...

0
Q: How would I read this sum. Convolution sum appearing in Forurier Galerkin methods.

MagnarI'm wondering how to read this sum: $w_k = \sum_{p+q=k} u_p v_q$

0
Q: Problem applying B.C.T in proof.

User1 I cant see which version and how they use Baires theorem to get that atleast on $MB_{n}$ is dense in some open set. Any version of Baires theorem needs open or closed sets. I can get neither on the $MB_{n}$'s.

Title contains problem. Problem applying B.C.T in proof.
0
Q: Sharp steps is commutative algebra exercise

erfanSuppose $C[0,1]$ is the ring of all real valued continuous functions and if $M $ is a maximal ideal of $C[0,1], i want show that the following set is nonempty. $$\{a \in [0.1] \| \forall f \in M , f(a)=0) $$

 
10:30 AM
0
Q: May I ask a question about logic written in the Prolog language on CS.SE?

Marine1Good morning! I'm learning logic through Prolog and I was wondering how to manage a Prolog resolution. I have an exercise to run by hand and to verify on a compiler but I don't know if Computer Science.SE is the right place to ask... I already asked it on Stack Overflow but didn't had any answer...

 
0
Q: Try to obtain the exponent of a variable in Maple.

LJRAre there some function $f$ in Maple such that $f(K[1,2]^(1/3)) = 1/3$? Any help will be greatly appreciated!

0
Q: Poisson process with circles

Oscar Consider a collection of circles in the plane whose centers are distributed according to a spatial Poisson process with parameter $\lambda|A|$, where $|A|$ denotes the area of the set $A$. The radius of each circle is assumed to be a random variable independent of the location of the center of...

0
Q: Closed convex cone for symmetric matrices

mlakhalCould some one please explain to me this remark: Let $S_{+}^{n}$ denote the set of positive semidefinite (psd) $n × n$ symmetric matrices. We write $X \succeq 0$ to denote that $X$ is symmetric and positive semidefinite. $\textbf{Remark} $...

Consider adding a tag for a broader subject area to which the question belongs. Some of these tags might fit. (from a bot)Normal Human 34 secs ago
 
0
Q: Is M.SE the right forum to ask question about logic written in Prolog language?

Marine1Good morning! I'm learning logic through Prolog and I was wondering how to manage a Prolog resolution. I have an exercise to run by hand and to verify on a compiler but I don't know if M.SE is the right place to ask... I already asked it on Stack Overflow but didn't had any answers... I am wonde...

 
0
Q: Inverse Fourier Transform of $| cos[(2 pi f)/100)] |$

NickI made 2 approaches Am I any close...how do i procced?

Short question. [Inverse Fourier Transform of $| cos[(2 pi f)/100)] |$](math.stackexchange.com/q/1571976)
0
Q: A problem about invertibility in an unitarian ring

Adrian BerteanuLet (R,+,*) be an unitarian ring. (a) If A is an unitarian ring. a,b are from R and a*b is invertible,it results that a is invertible and b is invertible? (b) If a is from R and a^n=a * a*...* a(n or) is invertible,it results that a is invertible? (c) If a is invertible to left and it isn't ...

0
Q: Prove 'every Positive Definite Matrix is Symmetric BUT not the other way around'

CypherI'm studying Linear Algebra and I read that : "Every Positive Definite Matrix is Symmetric BUT the vice versa is not correct..." in the text it suggests using 'x= u +iv' and then prove it by calculating (x,Ax). how can I prove it this way ? Thank you.

This site uses MathJax formatting of formulas. More tips here. (autocomment)Normal Human 26 secs ago
0
Q: Help solving a trig equation involving secants, cosecants, cotangents, and tangents

lightweaverIf I’m given $0=sec(x)tan(x)-csc(x)cot(x)$ in the domain $(0,\frac{π}{2})$, how would I algebraically find the value of $x$? I know the answer is $\frac{π}{4}$ because that’s where sine and cosine are equal, but algebraically, how would I properly find the answer?

0
Q: Fuzzy Logic from Many-Valued Logic?

HalabyHow can Fuzzy logic be derived from propositional many-valued logic (as a special case)?

 
10:59 AM
0
Q: Unexpected loss of reputation

Peter SmithI made a comment earlier today on a post requesting more detail. The question was removed and I lost two reputation points. I am puzzled. How can I lose points for requesting an improvement to a question?

 
0
Q: How to solve y' + y^2 - 2ysinx + sint^2x = cosx

SergeiHow to solve the following equation? y' + y^2 - 2ysinx + sint^2x = cosx It is necessary to determine the type and total solution. Help me please.

Short question. Question contains please. How to solve y' + y^2 - 2ysinx + sint^2x = cosx
0
Q: Finding a conditional distribution

john alexLet $U$, $V$, $W$ gaussian random variables $N(0,1)$ and $Z=\frac{U+VW}{\sqrt{1+W^2}}$. How can i find the conditional distribution of $Z$ knowing $W$ ?

 
11:28 AM
0
Q: How to solve (a+b)^1/2

user32091Hey I was wondering if it's possible to solve (a+b)^1/2. To give an example, (a+b)^2 is a^2 + b^2 + 2ab. But what is (a+b)^1/2? I have learned about binomial theorem and I still can figur it out. thanks in advance

0
Q: How to solve this type of recurrences?

BarrackRomneyg(x,y) = g(x-y+1,y) + g(x-y+1,y-1) I have dealt with equations where x and y don't interfere with each other like g(x,y) = g(x-1,y-1) + g(x-1,y) But here the x-y+1 term has been giving problems for determining a matrix for the recurrence.

Welcome to Math.SE, user32091. This site uses MathJax formatting of formulas. Questions tend to get more attention when they have a tag for a broad area of mathematics relevant to the question. Some of these tags might fit. More tips here. (from a bot)Normal Human 42 secs ago
0
Q: Feynman-Kac formula

user298078Let $u:[0,T]->R$ and $g:R->R$ positive,bounded and continuous functions. Let $u'(t,x)+(1/2)u''(t,x)-u^{1+\alpha}(t,x)=0,(t,x) \in [0,T]xR$ with u(.,T)=g Write u as a backward stochastic differential equation. In the correction, there is : With the Feynman-Kac formula, $u(t,x)=Y_t$ $Y_r=g(x+B_...

Short title. Feynman-Kac formula
 
11:46 AM
0
Q: Can I ask a question about Researchgate social network?

SSimonSome members expressed concerns and criticized RG network, is it alowed to open special topic about Researchgate, and to discuss about this problems? Example: There is actually an old ResearchGate question about this suggestion. – agold yesterday Why would you prefer ResearchGate? They have a ...

 
0
Q: Question on Divergence Theorem

The.SharkEvaluate ∫ ∫ F.ds. Where F= 59/3 x^3 i + 59/3 y^3 j + 59/3 z^3 k, and S is the surface: S= {(x,y,z) | x^2 + y^2 +z^2 =9} Please explain in detail how to get the answer.

Welcome to Math.SE, The.Shark. Words such as question are uninformative in titles. Please edit the title so that it better describes the specifics of your question. Do not hesitate to make it longer or include a formula if needed. This site uses MathJax formatting of formulas. More tips here. (autocomment)Normal Human 36 secs ago
0
Q: Is this (tricky) natural deduction with De Morgan's laws correct?

user3135672Just a practice question, however just wondering if this ND proof is correct? I have put brackets in 2.2 and not in 2.3 however this shouldn't make a difference?

0
Q: Distribution of method of moments estimate

Sung Pang$X_{1}, ... , X_{n}$ $i.i.d$ with density function $$ f(x|Θ) = e^{-(x-Θ)}, x \ge Θ$$ Method of moment estimate $\hat{Θ_{MME}}$ of Θ : $$\hat Θ = \bar{X}-1$$ How can I find the distribution of MME ? I found MLE, distribution of MLE, MME but I can't find distribution of MME. Please answer..

0
Q: Deformation retracts to a point.

MihailIf space $X$ deformation retracts to a point $x\in X$, then for each open $U\in X$ containing $x$ there exists an open $V\in U$ again containing $x$ s.t. inclusion of $V$ into $U$ is nullhomotopic. My attempt: Since $X$ deformation retracts to a point $x$, there is a corresponding map $F:X\times...

Tagged proof-verification. Deformation retracts to a point.
0
Q: Roots of: $2(x+2)(x-1)^3-3(x-1)^2(x+2)^2=0$

RaphaelThis is mybe a stupid quastion, but I want to the roots of: $$2(x+2)(x-1)^3-3(x-1)^2(x+2)^2=0$$ What that I did: $$\underbrace{2(x+2)(x-1)(x-1)(x-1)}_{A}-\underbrace{3(x-1)(x-1)(x+2)(x+2)}_{B}=0$$ So the roots are when $A$ and $B$ are both zeros when $x=1$ and $x=-2$ My questions: $1)$ ...

 
12:30 PM
0
Q: How is this possible? $\int_{0}^{\infty}xe^{\theta-x}dx=\theta+1$

Bozo VulicevicHow is this possible? $$\int_{0}^{\infty}xe^{\theta-x}dx=\theta+1$$ I even used a program online and the answer is $e^{\theta},$ could this be a mistake?

0
Q: i have a little question ...

Mohammed Jamalsup all i need a method to find all subgroups from any finite group for example the subgroup of d4 (ORDER 4 ) {(1),(1.2.3.4),(1,3)(2.4),(1.4.3.2.)} okay i can understand this but the others .. how we can get them ?? there is any theorem or somethin?

0
Q: $f:(a,b) \to \mathbb R$ be continuous such that $|f|$ is differentiable in $(a,b)$ ; then is $f$ differentiable in $(a,b)$ ?

Saun DevLet $f:(a,b) \to \mathbb R$ be a continuous function such that $|f|$ is differentiable in $(a,b)$ ; then is $f$ differentiable in $(a,b)$ ?

Welcome to Math.SE, Mohammed Jamal. Words such as question are uninformative in titles. Please edit the title so that it better describes the specifics of your question. Do not hesitate to make it longer or include a formula if needed. More tips here. (autocomment)Normal Human 1 min ago
0
Q: I have 2 equations Y(wa,wb) and Z(wa,wb), how could I get wa(Y,Z) and wb(Y,Z)?

invitadoThe basic question is, If I have y as function of x (y=x^2 for example), How do I get x as function of y(x=sqrt(y) for example)? I have that problem but with two equations: y=wa^2+wb^2 z=1-y=1-(wa^2+wb^2) I want wa as function of y and z, and wb as function of y and z. But I don't know how to...

This site uses MathJax formatting of formulas. More tips here. (from a bot)Normal Human 49 secs ago
0
Q: finding a regular length

annai am trying to find a regular language that: her minimum pumping length = number states of minimum Nondeterministic automata = number of equivalence classes. (not 1) any ideas? Thanks in advance

 
12:52 PM
1
Q: What happens if you fail the review audit several times in a row?

AniketWhat happens if you fail the review audit several times in a row? Do you get suspended from reviewing? I have failed 4 review audits in the close vote category in the last few days and since today I cannot find any close vote questions in the queue. Am I suspended?

 
0
Q: How to show that a function is strictly decreasing.

James2890The function $f(x)$ is defined by $f(x)=\frac{x^2+2x}{x^2-1}$. How would you show that $f(x)$ is a strictly decreasing function.

0
Q: estimate of n factorial

MatFyzakon our lesson at our university, our professsor told that factorial has thie estimates $n^{\frac{n}{2}} \le n! \le \left(\dfrac{n+1}{1}\right)^{n}$ and during proof he did this $(n!)^{2}=\underbrace{n\cdot(n-1)\dotsm 2\cdot 1}_{n!} \cdot \underbrace{n\cdot(n-1) \dotsm 2\cdot 1}_{n!}$ and then...

0
Q: Circle of radius of Intersection of Plane and Sphere

MathematicsThe plane $x+2y-z=4$ cuts the sphere $x^2+y^2+z^2-x+z-2=0$ in a circle of radius? I tried putting value of y from plane in sphere but then I get a $zx$ term. How to proceed?

0
Q: Exponential of a Jordan block

Suleyman OrazgulyyevI am having difficulties with calculating exponential of a Jordan block, I cannot understand the method, can please someone help me, I have an exam on Monday. 'J' is my Jordan matrix and 'P' is my eigen vector matrix, 'A' is my starting matrix that I obtained 'J' from, I will also give you the r...

0
Q: compute integration $\int_{0}^{2\pi}sin^3(3e^{iz}+\frac{\pi}{4})dz$

user296431If we assume z is a complex number can you compute the following complex integral $\int_{0}^{2\pi}sin^3(3e^{iz}+\frac{\pi}{4})dz$ I tried to change the variable putting $u=3e^{iz}+\frac{\pi}{4}$, but i couldnt proceed. Thanks for your help.

 
1:37 PM
0
Q: iterated unprovability

D.F.FHi all the following might be a silly question it is well known that some statements like for example CH are not provable within ZFC (assuming consistency of course) ie. $ZFC\not\vdash CH$. However, CH is not so bad in the sense that at least its unprovability is provable ie. $ZFC\vdash \ulcorne...

0
Q: Quantized random vector

souravConsider system to be: $\bar y= W\bar h+ \bar v$ where, $\bar v$ is Gaussian random vector with mean zero and covariance $R_{v}$ So, from $\bar y $ and the estimated vector $\hat y$, error is determined and quantized to have a vector $e$ of +1 and -1. I wish to determine $p(\bar b / \bar h )$...

0
Q: Replicate Matlab integrator block in MS excel

deduuI created a simple diagram to solve ordinary differential equation as shown below. Simple ODE I was trying to compute the result of xf_dot manually in Ms Excel but I did not get the same answer with the solution from Matlab. The xl_dot is a table of time-value e.g ([0 30; 1 27; 2 24; . . .])...

Welcome to Math.SE, deduu. This site uses MathJax formatting of formulas. More tips here. (from a bot)Normal Human 23 secs ago
0
Q: laplacian of a graph

Daniel I'm following the first lecture of Daniel Spielman's lecture notes on Spectral Graph Theory (http://www.cs.yale.edu/homes/spielman/eigs/lect1.pdf). It begins with the following example of a graph G=(V,E): (1) --- (2) --- (3) Where V={1,2,3} and E={(1,2),(2,3)}. The author says that he compute...

Short title. laplacian of a graph
0
Q: How to see this inequality

RhjgConsider $$ \frac{dx}{dt}=\sqrt{x^2+1}+t^2,\qquad x\in\mathbb{R}. $$ Then I do not see why it is $$ \lvert\frac{dx}{dt}\rvert\leq 2\lvert x\rvert+t^2. $$

Short title. Short question. How to see this inequality
0
Q: Analytic functions and constants question

CaydeLet $f(z)$ and $g(z)$ be analytic on some domain. Show that if $Re(f(z)) = Re(g(z))$ then $f(z)-g(z)$ is constant.

Words such as question do not add information to titles. Please edit the title so that it better describes the specifics of your question. Do not hesitate to make it longer or include a formula if needed. More tips here. (from a bot)Normal Human 24 secs ago
0
Q: Nowhere locally integrable

yadaddyDoes there exists a function $f : I \to \mathbb{R}$ defined on an interval $I \subseteq \mathbb{R}$ that is measurable but nowhere locally integrable, i.e. not integrable on any compact subinterval $[a,b] \subseteq I$? One can try to call for Lusin's continuity theorem: If $m$ denotes the Lebesg...

0
Q: prove by induction solving

zeeksI had the following prove by induction problem in an exam and I didn't do it because I didn't know how to. Could anyone solve it, please? $F(0) = 0$ $F(1) = 1$ $F(n) = F(n-1) - F(n-2)$ $F(n) <= (\frac{1+\sqrt{5}}{2})^n$ Thank you

Short title. Question contains please. prove by induction solving
0
Q: Laplace Equation Dirichlet Problem on a disk in the plane

GabrielIf D be the unit disk then what could be the solution of a problem Δu=0 in D,∂u/∂n=cos3θ on ∂D I have tried using the polar co-ordinates representation to solve this but the boundary seems to be contradicting.

 
1:59 PM
-1
Q: Stack Overflow at Its Finest

IkeWhat kind of image do people conjure up with this title, of SO at its very finest? For me this is an example: How to determine whether my calculation of pi is accurate?, where the world record holder of computing pi ended up answering a question related to computing pi. I was trying to put a fi...

 
0
Q: Calculate the variance and expectection of $\hat{y}$ in a linear regression model

nbroI have the following linear regression model $$y = \beta_0 + \beta_1 \cdot 40$$ and I would like to calculate the expected value and the variance, but I am not sure how to do it, even having the rules for computing the expectation and variance. From my slides $$E[y] = 51.7590$$and $$Var[y] = 18...

0
Q: $1-z^p\leq \left(1-z\right)^p$ for $0<p\leq 1$ and $0\leq z \leq 1$

Kamal SalemI need to proof an inequality , if $0<p\leq 1$ and $0\leq z \leq 1$, then $1-z^p\leq \left(1-z\right)^p$ Can be see Holder continuity of power function

0
Q: In Need of Logarithms Simplification Exercises

ZybergI am very interested in mathematics, however, finding nowhere near wanted information in school sometimes I go and learn something by myself. Just like this time. I decided to learn more about logarithms as they always picked my interest, even thought it's a year or two more advanced than I shoul...

Welcome to Math.SE, Zyberg. This site uses MathJax formatting of formulas. More tips here. (autocomment)Normal Human 25 secs ago
 
2:17 PM
0
Q: To prove that a vector x(t) lies in a plane.

Raj KumarProve that vector x(t)=ti+(1+t/t)j+(1-t^2/t)k lies in a curve. I am puzzled. Don't know how to approach it.

0
Q: Limit including 2 variables

Rahul KambleWhy $\lim_{x\to a}\{\lim_{y\to b}\f(x,y)}$ \neq $\lim_{y\to b}{\lim{x\to a}\f(x,y)}\$?

0
Q: Some basic questions regarding rank-1 matrices,

User001If an $nxn$ matrix $B$ has rank 1, and A is another $nxn$ matrix, then why does $AB$ also have rank 1? This showed up in a solution that I read through, but it doesn't seem like an obvious fact. And one more thing that came up in this solution: it says that since this matrix has rank 1, then i...

Short title. Short question. Limit including 2 variables
0
Q: Continuity of maps

VrouvrouWe consider this topological space $(\mathbb{R},\tau)$ where: $$\tau=\{G\subset \mathbb{R}, (\mathbb{R}\setminus G)~\text{countable}~\}\cup\{0\}$$ and we consider the identity map $$f: (\mathbb{R},\tau)\rightarrow (\mathbb{R},|.|)\\~~~ x~~~\mapsto ~~~ x$$ How to prove that $f$ is continuous usi...

Short title. Continuity of maps
0
Q: Find the form of $f$

JaneFor $f: \mathbb{R} \rightarrow \mathbb{R}$ , the following holds: $\forall x,y \in \mathbb{R} : f(x+y) = f(x)\cdot f(y)$ $\forall y : \lim_{x\to y}f(x) = f(y)$ $f$ is not identically $0$ Find the form of the function (rational/exponential/logarithmic/other...) I was thinking this: if $f(x) = y$ f...

Short title. Find the form of $f$
0
Q: Integral of (sine e^x dx) ??

David 2000I was working on a physics problem that I faced with this Integral : $$\int sin(e^x) \, dx =$$ I tried a lot to solve it but I could not.

Short title. Title contains ??. Short question. Integral of (sine e^x dx) ??
 
2:37 PM
-1
Q: Downvoting for code not added properly into the question area

coderI have been using Stack Overflow from since 4 years, but haven't not got the idea of how to add huge lines of code for SQL into the question area. On contrary of that, I have been getting downvote just for not adding the code in a proper format. Is this right? Since, if we get a downvote from a...

 
0
Q: Spectrum of sum is subset of sum of spetrums

kp9r4dLet $A$ - unital banach algebra, $a,b \in A$, I want to proof that $\sigma (a+b) \subset \sigma(a) + \sigma(b)$. How can I do that?

0
Q: Function that satisfies heat equation

GabrielI have been asked to prove the below function satisfies heat equation. u(r,t)=2Uπ∑n=1∞ {(−1)^n+1}(1/r)sin(nπr)e^−n^2π^2t I have tried to equate ∂u/∂t = ∂^2u/∂x^2 however both let hand side and right hand side are not tallying. Can you please help. Is this the correct function?

This site uses MathJax formatting of formulas. Consider adding a tag for a broader subject area to which the question belongs. Some of these tags might fit. More tips here. (from a bot)Normal Human 53 secs ago
0
Q: Calculate connecting line and circular arc between two points and angles

Ge WeGiven two points and angles, how can I calculate the connection between the two points in the form of a straight line and a circular arc? Context of the question: I am trying to make a program that converts a model railway layout created in XtrkCAD into a SCARM layout (scarm.info). Both design a...

0
Q: If $P \lor Q$ and $\neg P \lor \neg Q$ are both true, do we get a contradiction?

Arnie DrisQuestion If $$P \lor Q$$ and $$\neg P \lor \neg Q$$ are both true, do we get a contradiction? My Attempt Since $$\left\{\neg P \lor \neg Q\right\} \Longleftrightarrow \neg \left\{P \land Q\right\}$$ the question essentially asks whether $$P \lor Q$$ is logically equivalent to $$P \land Q$$ wh...

Questions tend to get more attention when they have a tag for a broad area of mathematics relevant to the question. Some of these tags might fit. (from a bot)Normal Human 1 min ago
0
Q: About linear maps - is this true?

user118224I've been thinking about this. My the reasoning seems to imply that: If two linear maps $f,g$ disagree somewhere, they must disagree everywhere but $0$. This feels like it can't be right, but I can't see a fault in the reasoning: Say $f,g$ linear on $V$ disagree at $x_0$. For any $x\neq...

Tagged proof-verification. About linear maps - is this true?
 
0
Q: Is Math.SE lacking in the area of Stochastic Processes?

An old man in the sea.At the moment I'm reading some texts on Markov Chains, to learn the subject, and I'm getting the feeling that Math.SE community is not interested/very knowledgeable in this area, when compared to others more theoretical. Why is that? I do not intend to disrespect anyone, with this question. Che...

 
0
Q: $\lim_{x \to 0}\frac{f(3x)}{\ln(1+4x)} = 2.25$

Natalia$f$ is defined on the neighborhood of $x=0$, $\lim_{x\to 0}\frac{f(x)}{x} = 3$. I need to prove that $\lim_{x \to 0}\frac{f(3x)}{\ln(1+4x)} = 2.25$. I'm kinda stuck. I was thinking:If I define $t = 4x$ then $\lim_{x \to 0}\frac{f(3x)}{\ln(1+4x)} = \lim_{t \to 0}\frac{\frac{f(3 \cdot \frac{t}{4})}...

Welcome to Math.SE, Natalia. A title should not be all-MathJax; having some plain text helps with search and navigation. (autocomment)Normal Human 24 secs ago
0
Q: Finding a vector perpendicular to two other vectors

TheNotMeSuppose we are in $R^n$. Mark $v_1 = 1^n$, the all $1$'s vector. Set $v_2(i) = n+1-2i$. How do I find a vector that is perpendicular to both of them?

0
Q: Circle homography

Ami HollanderI'm attending a 3d-graphics course and I want to figure out which homograpic transformations conserve a circle's equation. The circle's equation is given as: Circle = x^2 + y^2 + Ax + By + C = 0 So in an effort to find out some transformations, I though about creating a matrix H={h1,h2,h3;h4,...

Welcome to Math.SE, Ami Hollander. This site uses MathJax formatting of formulas. More tips here. (autocomment)Normal Human 55 secs ago
0
Q: Average speed question (puzzle)

Jean PaulCan someone help me with this puzzle problem: James walked for two days. On the second day he walked 2 hours longer and with a average speed of 1 km/h faster then he did on the first day. On the second day he walked 64 km in 18 hours. What was his average speed on the first day?

Words such as question are uninformative in titles. Please edit the title so that it better describes the specifics of your question. Do not hesitate to make it longer or include a formula if needed. More tips here. (autocomment)Normal Human 21 secs ago
0
Q: please help me solve this really disturbing question

le Professeur Integrate the following function wrt dx \int 1/(1+x^7)

Welcome to Math.SE, le Professeur. Words such as please, help, question do not add information to titles. Please edit the title so that it better describes the specifics of your question. Do not hesitate to make it longer or include a formula if needed. More tips here. (from a bot)Normal Human 28 secs ago
0
Q: if $f(x) = x+x^3$, then what's $f′(x)$?

David 2000I was working on a math problem that I faces with this : R→Rf:R→R $f (x) = x + x^3$ then what's $f′(x)$

0
Q: Convergence of supremum norms

User1110192101I'm studying the difference between pointwise and uniform convergence for sequences of continuous functions. Is it true that if $f_{n} \to f$ pointwise, then $ \|f_{n}\|_{\infty} \to \|f\|_{\infty}$? It seems true when they converge uniformly, but I can't find a counterexample if the convergenc...

 
3:30 PM
0
Q: How to solve this first order differential equation?

JuemblesSo D.E are pretty new to me but i have made some linear ones but i cant seem to get how to solve a non-linear one. (2x+3y+4)dx+(3x+4y+5)dy=0

Welcome to Math.SE, Juembles. This site uses MathJax formatting of formulas. More tips here. (from a bot)Normal Human 32 secs ago
0
Q: Chess tournament problem

algebra112 chess players took part in a tournament. Each played against each other exactly once. After the tournament did every chess player 12 lists of names. On the first list, the player only wrote his own name. On the second list, they wrote their own names as well as all man they had won against. Th...

Welcome to Math.SE, algebra1. Tag (problem-solving) should not be the only tag a question has. Please add a tag for a subject area to which the question belongs. (autocomment)Normal Human 31 secs ago
0
Q: Prove that each closed cycle in G has a minimum length of 5

HasGiven Graph G How can I prove that each cycle in G has a minimum length of 5?

Title ends with a digit. Short question. Prove that each closed cycle in G has a minimum length of 5
 
3:46 PM
-1
Q: Why are so many people on ths site so demeaning and rude to people who ask questions?

CJGI realize that this is not a coding question but I really think it needs to be addressed. This site was created for people to come and ask questions... and why do people ask questions? because they need help. I constantly see people putting others down for asking stupid questions or saying things...

 
0
Q: How to calculate the limit of (2^n + (-1)^n)/(2^(n+1) + (-1)^(n+1)) as n goes to infinity?

RogetI need to know the way and how to calculate the lim (n->infinity) of (2^n + (-1)^n)/(2^(n+1) + (-1)^(n+1)) but i'm clueless...

Welcome to Math.SE, Roget. This site uses MathJax formatting of formulas. More tips here. (from a bot)Normal Human 39 secs ago
0
Q: Sum of Treble trigonometric Series

Samith AbeywickramaI am very much thankful to you if you can help me to find summation of following series. \sum_{i=1}^{N-2} \sum_{j=i+1}^{N-1} \sum_{k=j+1}^{N} [\sin (ix-jx) + \sin (jx-kx) + \sin (kx-ix)]

Welcome to Math.SE, Samith Abeywickrama. This site uses MathJax formatting of formulas. More tips here. (autocomment)Normal Human 25 secs ago
0
Q: Help with precentage

queens718Can someone please explain how to calculate "percent more than" and "percent less than"? I know 35 is 75 percent more than 20 - but no idea how to calculate it. Also trying to figure out how to find percent less than for: 120 is what percent less than 200? Thank you!

Words such as help do not add information to titles. Please edit the title so that it better describes the specifics of your question. Do not hesitate to make it longer or include a formula if needed. More tips here. (from a bot)Normal Human 28 secs ago
0
Q: Inverse of cosine

FlameI got a question I think is fairly easy to answer but I cant get my head around it: cos(45 deg) = 0.7071... This is the point on the x-axis. How do I go from a value like 0.7071... to the amount of degrees?

Short title. Inverse of cosine
0
Q: if $f(x)=x+x^3$ then what's $f^-1(x)$

mihiI was studying about functions that I faced with this $$f:R→R f(x)=x+x^3$$ then what's $f^-1(x)?$ I tried a lot but I could not solve it

0
Q: Finite Cartesian Product of Countable sets is countable?

An old man in the sea.Is the finite Cartesian Product of Countable sets countable? If possible, could you give a bijection that would work for any example? Any help would be appreciated.

 
4:15 PM
1
Q: When is asking a duplicate ok?

JosephThis question: Kinetic theory derivation of viscosity of a gas has an accepted answer. That answer, however, does not contain the amount of detail that I want for a specific part of the question. I have (in the recent past) placed a bounty on the question asking for more detail, however no other ...

 
0
Q: $\color{teal}{\large\text{Task assignment}}$

Math FreakIn how many ways can you assign $\:\eta\:\:$different integer tasks $\left(\:\eta<\infty\:\right)\:$to $\:k\:\:$employees $\left(\:k<\eta\:\right)$ if the toughest task must belong to your best employee, whilst the simplest task shall be executed by the only 2016 newcomer of your firm? $\\$ ...

Words such as task do not add information to titles. Please edit the title so that it better describes the specifics of your question. Do not hesitate to make it longer or include a formula if needed. A title should not be all-MathJax; having some plain text helps with search and navigation. More tips here. (autocomment)Normal Human 25 secs ago
0
Q: How do I integrate this one?

Luca MuscoloIntegral of: this, sorry gentlemen I'm still unaware how to write formulas , ( è - é)

Short title. Short question. How do I integrate this one?
0
Q: Relaxed condition for differentiability - proof check!

user118224I am trying to show that if the two partials of $f:\mathbb{R}^2\to\mathbb{R}$ exist at point, it is enough for only one to be continuous there to imply that $f$ is differentiable there. To show this I have decided to prove that: $f:\mathbb{R}^2\to \mathbb{R}$ not differentiable at $(a,b)$. If...

0
Q: Uniqueness of the real line

MickGA few days ago, I came across this question in a review queue. I tried my luck at it. Here is what I did: If I want a homomorphism (isomorphism, but even just homomorphism) $f:\mathbb{R}\to F$, then I'll need 0 and 1 to map to themselves. From this, via homomorphism properties, I conclude $\mat...

-1
Q: A definition in cohomology

1234 I am looking for the definition Tangent cohomology. I found some different definitions in text books

Short title. Short question. A definition in cohomology
0
Q: Suppose $f: [a,b] \rightarrow \mathbb{R}$ is Riemann intergrable on $[a+\epsilon, b]$ for all $0<\epsilon<b-a$.

bettybSuppose $f: [a,b] \rightarrow \mathbb{R}$ is Riemann intergrable on $[a+\epsilon, b]$ for all $0<\epsilon<b-a$. Then $f$ is Riemann integrable on $[a,b]$?

Short question. [Suppose $f: [a,b] \rightarrow \mathbb{R}$ is Riemann intergrable on $[a+\epsilon, b]$ for all $0<\epsilon<b-a$.](math.stackexchange.com/q/1572340)
0
Q: Hodges exercise 2.7.1: Quantifier elimination in dense linear orderings

elWandereroIn Hodges' A Shorter Model Theory, exercise 2.7.1 tells you to prove theorem 2.7.1, which says that the following five formulas are an elimination $\Phi$ set for the class of all dense linear orderings (with a signature consisting only of "$<$"): There is a first element There is a last element...

Consider adding a tag for a broader subject area to which the question belongs. Some of these tags might fit. (autocomment)Normal Human 22 secs ago
0
Q: Kernel, range basis

pavelI'm preparing for my final, and i'm a bit confused with kernel,range...Please check my conclusion about the topic can you help me? basis of matrix = number of pivots after r.r.e.f. (When i'm finding basis, should I add one more column with zeros into the matrix?) Dimension = number of columns ...

Short title. Kernel, range basis
0
Q: Find the solution for this differential equation

MathematicsSolve the differential equation; $(xdx+ydy)=x(xdy-ydx)$ L.H.S. can be written as $\frac{d(x^2+y^2)}{2}$ but what should be done for R.H.S.?

 
4:57 PM
0
Q: Cone of an adjunction

AnalysisStudent0414I came across this sentence "...let $\varepsilon: GG^\vee \to Id$ be the counit of adjunction and $Z$ its cone." I thought that cones were constructions on functors. $\varepsilon$, though, is a morphism of functors (a natural transformation)... in this context, what does "its cone" mean? Or is...

0
Q: Seeing a pattern (puzzle)

Jean PaulDoes someone see a pattern?: 1 11 21 1211 111221 312211 13112221 1113213211 31131211131221 13211311123113112211 111312211331121321113212221

 
5:11 PM
0
Q: Can somebody help me to proof problem of topology?

user298135A is the convex subset of R^n. Proof, that Pi(A, a) group is trivial : Pi(A, a) = 0, where a is fixed point from A. Thanks.

Welcome to Math.SE, user298135. Words such as somebody, help do not add information to titles. Please edit the title so that it better describes the specifics of your question. Do not hesitate to make it longer or include a formula if needed. More tips here. (autocomment)Normal Human 28 secs ago
-1
Q: Show that the following linear transformation is normal.

Vitale Show that if $\|Ax\|=\|A^{*}x\|$ for all $x$ beloging to the finite-dimensional inner product space $X$, the linar transformation $A$ is normal.

0
Q: Prove that $\log x<\sqrt{x}$ for $x\geq 1$

EpsilonDelta Prove that $\log x<\sqrt{x}$ for $x\geq 1$ Let $f(x)=\sqrt{x}- \log x$. So, $f(1)=1>0$. $f'(x)=\frac{1}{2\sqrt{x}}-\frac{1}{x}>0$ only when $x>4$. When I draw the graph of $f$ in Wolframalpha, it shows the result, but how do I prove it rigorously? Can someone please help?

0
Q: Interpreting results of all_node_cuts (looks incorrect to me)

user298139I am utilizing the all_node_cuts function to get all collections of two nodes that disconnect the graph if removed (link below). https://networkx.github.io/documentation/latest/reference/generated/networkx.algorithms.connectivity.kcutsets.all_node_cuts.html Here is how the undirected graph look...

Welcome to Math.SE, user298139. This site uses MathJax formatting of formulas. More tips here. (autocomment)Normal Human 31 secs ago
0
Q: Is it Linear Transformation or not?

abdul rabT: R^2 ---> R^2 T(x,y)=(x^2,y) Is it Linear Transformation or not?

0
Q: Is p(x) reducible over the rings R?

BananaIs $p(x) = x^4+2x-2$ reducible in any of the following rings: $\mathbb{Q}, \mathbb{R}, \mathbb{C}, \mathbb{F_9}$ ? How do you show this for all the individual rings?

0
Q: Why can't I do this?

Kraig Lee=dollar(if(s11>0;(s11*e11)+1;e11-k9&" more to buy")) I'm trying to display currency format for a number and text only when outcome is false: ??It brings back some error?? I'm using OpenOffice v4.1.1 You can put the formula as =dollar(if(s11>0;(s11*e11)+1;e11-k9))&"enter". It will display "ent...

Short title. Why can't I do this?
0
Q: How can I convert a Trapezoidal membership function to a triangular membership function??

All informationHow can I convert a Trapezoidal membership function to a triangular membership function??

Welcome to Math.SE, All information. Questions tend to get more attention when they have a tag for a broad area of mathematics relevant to the question. Some of these tags might fit. (from a bot)Normal Human 27 secs ago
0
Q: Geometry related problem

Angel MarinaD is a point on the circle that passes through the point A, B, C. In triangle ABC, AB=4, BC=1, ∠ABC=120.The maximum area of ABCD can be written as 'a*sqrt(c)/b' , find the value of a+b-c?

Welcome to Math.SE, Angel Marina. Consider adding a tag for a broader subject area to which the question belongs. Some of these tags might fit. (from a bot)Normal Human 40 secs ago
0
Q: Help setting up double integral over a general region

Jared M.How do I evaluate the following integral $$ \int_{-1}^1 \int_{arccos(y)}^{\pi} {sin(x)\sqrt {1+{sin^2{x}}}}{dxdy}$$ I attempted to change the limits for the integrals $$ \int_{0}^{\pi} \int_{cos(x)}^{1} {sin(x)\sqrt {1+{sin^2{x}}}}{dydx}$$ which I can integrate in respect to y giving $$ \int_{0...

Words such as help do not add information to titles. Please edit the title so that it better describes the specifics of your question. Do not hesitate to make it longer or include a formula if needed. More tips here. (from a bot)Normal Human 30 secs ago
0
Q: Integral with difficult substitution

davoidI have a problem with this integral. I don't what kind of substitution use to solve it. Please help. int t*sqrt{(1-9t^4)}

Words such as difficult are uninformative in titles. Please edit the title so that it better describes the specifics of your question. Do not hesitate to make it longer or include a formula if needed. More tips here. (from a bot)Normal Human 25 secs ago
0
Q: A question on notation in convex optimization

cgo$\mu_E$ is a $4 \times 1$ vector composed of known constants, and $\mu$ is a vector of the same dimension but with unknown variables. Let us say $\mu = (x_1, x_2, x_3, x_4)^T$. What is the meaning of the following notation in optimization? minimize $|\mu_E - \mu|_2 $ subject to some constrain...

Words such as question are uninformative in titles. Please edit the title so that it better describes the specifics of your question. Do not hesitate to make it longer or include a formula if needed. More tips here. (autocomment)Normal Human 41 secs ago
1
Q: Why (Zn,*), integers modulo n under moltiplication, is a group if and only if n is prime?,*) is a group if and only if n is prime?

B.niaIs it true that (Zn,*), integers modulo n under moltiplication, is a group if and only if n is prime? If it's true, why? How can I prove it?

0
Q: Determinant multiplication (det(ab)=det(a)det(b))

Nesahttps://proofwiki.org/wiki/Determinant_of_Matrix_Product I found this proof (Proof 2).Could you tell me two things? 1: How do I know that if a matrix is invertible then it is a product of elementary matrices? 2: How do I see that if $A$ is not invertible then neither is $AB$?

0
Q: Partial surjective function

zakumI want to reduce $\overline{K}$ to the set of natural numbers such that the program codified by them computes some partial and surjective function, in order to prove that such set is not semi-decidable. Formalizing, we have: $$\overline{K} ≤ \{ p ∣ φ(p)\ is\ partial\ and\ surjective \}$$ Howe...

 
5:59 PM
0
Q: CSS Bug on the Review Page

Julian E.I think there may be a bug on the review page of SO, as the Footer floats in the middle of the page and leaves a big white gap underneath it. This was checked on an iMac 2014 27inch, running the latest Safari version. Below is a screenshot:

0
Q: How can we review or ask for review the single handled decisions?

Tareq MahmoodI just see a person has marked a question as duplicate single handledly since he has javascript badge and "These users can single-handedly mark javascript questions as duplicates and reopen them as needed." But what if he does a mistake? Is there any way I or any other user can ask for additio...

0
Q: An auto-comment on first questions that motivates sharing doubts and thoughts

HowDoIMathVery often I see a user's first question getting down-voted and eventually it ends up being closed. Usually I find this to be too rough of a treatment, as the user is not accustomed to the site. I know that the close votes are usually formally justified, if the user includes no effort on his or ...

 
0
Q: When an Emergent finite automata learns from a symbolic finite automata. Only patterns are used for state symbols. (True or False??)

GiorgioWhen an Emergent finite automata learns from a symbolic finite automata. Only patterns are used for state symbols. (True or False??) I'm confused if this is true or false? I want an explanation why as well? Thank you

 
6:12 PM
0
Q: prove integer linear programming decision problems are NP-complete .

Alishow that if traveling salesman problem decision problems are Np-complete then integer linear programming decision problems are also NP-complete.

Questions tend to get more attention when they have a tag for a broad area of mathematics relevant to the question. Some of these tags might fit. (autocomment)Normal Human 33 secs ago
0
Q: change variable in integrale?

kawazakii have this simple question : i want to change the variable here and put $u=nx$ then what wil be equal the integral (here we are in $\mathbb{R}^n$ $\int_{\mathbb{R}^n}|f(nx)|^qdx$ ($q$ real >1) thank you very much.

 
0
Q: Warnings about using Python's EXEC statement while trying to generate variable names

VynylynI was reviewing some discussions: Dynamically set local variable in Python Using a string variable as a variable name How can you dynamically create variables in Python via a while loop? where some people answered questions by using Python's exec statement. I noticed that whenever it's sugges...

 
0
Q: A Problem about Vector Spaces and Linear Maps

MJFIn my textbook the following is proposed as a theorem (with some explanation but without proof), but it seems wrong to me. Maybe it's an easily-fixable typo; I don't know. I will quote the (so-called) theorem and then point out its error. You may want to inform me if I'm wrong or mistaken (I'm pr...

0
Q: Field Trace Identities

AndrewLet $\mathbb{F}_q$ be the finite field with $p^n$ elements and consider the trace map $$\mbox{Tr}: \mathbb{F}_q\to \mathbb{F}_p,$$ where $$\mbox{Tr}(\alpha)=\alpha+\alpha^p+\alpha^{p^2}+\cdots +\alpha^{p^{n-1}}.$$ If $\varphi \in \mbox{Gal}(\mathbb{F}_q/\mathbb{F}_p)$, then $\mbox{Tr}\big( \varp...

0
Q: $A=\begin{bmatrix} sin (\pi/18) & -sin (4\pi/9) \\ sin (4\pi/9) &sin (\pi/18)\end{bmatrix} $ find number $n\in N$ such that $A^n=I$?

Cielo$A=\begin{bmatrix} sin (\pi/18) & -sin (4\pi/9) \\ sin (4\pi/9) &sin (\pi/18)\end{bmatrix} $ find number $n\in N$ such that $A^n=I$. I found eigen values and eigen vectors and use it to find the value of n but it is lengthy , is there another way of solving it.

0
Q: What am I doing wrong in this proof?

Andrew VThe question is this: Let $f:\mathbb{R}\to\mathbb{R}$ be differentiable at $x=0$ and suppose that there is a number $L$ such that $$\lim_{x\rightarrow0}\frac{f(x)-f(x/2)}{x/2}=L.$$ Prove that $f'(0)=L$. Here's my answer with all theorems referenced being from Rudin: Let $a_n$ be a positive se...

0
Q: determining Fourier series coefficients

Jeje Zzzf(x)= \begin{array}{cc} 2 & if0<x<2\\ -2 & if-2<x<0 \end{array} I found that all coefficient (a0,an,bn) is it correct?

 
6:37 PM
0
Q: User neither marking question nor votes up

GhayelI am spending 2-3 hours regularly on this website to help users but its pity most of the users neither marking answer nor voting up that disappoints. The users realized in many of my answers they got their solution but not marked my answer or voted up. Is there any rule for that How can I ed...

 
0
Q: Decide whether this is a ring

user297125Decide whether (Z × Z, +, ∗) is a ring. Operation + is defined by the following formula: (a, b) + (c, d) = (a + c, b + d). Operation ∗ is defined by: (a, b) = (ac + bd, ad + bc).

This site uses MathJax formatting of formulas. More tips here. (from a bot)Normal Human 25 secs ago
0
Q: Lifting Functors to Adjoints

SladeThere are well-understood theorems that give sufficient conditions for a functor $R: D\to C$ to have a left adjoint. For example, $R$ should preserve limits and $D$ should have nice categorical properties. But there are situations in which $R$ does not preserve limits, yet it lifts to a right a...

0
Q: Proof: Is there a line in the xy plane that has all rational coordinates?

MatthewQuestion: Is there a line in the xy plane that has all rational coordinates. Prove your answer. Idea: There is most certainly not. I believe it can be shown that between any 2 rational points that there is at least one rational coordinate. Therefore, there can not be a line that contains only r...

0
Q: Number of Combinations?

StatsGuruNeed to select one winner from each of three matches that each have three options. Game One: Home or Visitor or Tie Game Two: Home or Visitor or Tie Game Three: Home or Visitor or Tie How many different combinations are there? Appreciate any help!

0
Q: covariance of a point between 2 points with known covariances

mjp2711If two points P1 and P2 observed in a plan are each associated with a known covariance matrix, is it possible to infer the covariance matrix of any point on the segment [p1,p2]?

 
00:00 - 19:0019:00 - 00:00

« first day (40 days earlier)      last day (534 days later) »